PCC Exam III

Ace your homework & exams now with Quizwiz!

While the nurse is obtaining the health history of a 75-year-old female patient, which of the following has the greatest implication for the development of cancer? a. Being a woman b. Family history of hypertension c. Cigarette smoking as a teenager d. Advancing age

ANS: D According to the American Cancer Society, the most important risk factor for cancer development is advancing age.

The nurse is preparing to administer medications to a patient with rheumatoid arthritis (RA). The nurse should explain which goal of treatment to the patient? a. Eradicate the disease b. Enhance immune response c. Control inflammation d. Manage pain

ANS: C Medications for RA are intended to control the inflammation that results from the body's hyperimmune response. Autoimmune diseases like RA are chronic and currently have no curative treatments. Autoimmune diseases like RA are caused by hyperimmune response. The immune system needs to be suppressed, not enhanced. While the medications used for RA might help with pain management, the goal of medication intervention is to manage the inflammation. REF: Page 223 - Page 224

HIV infects and destroys CD4 cells. List the following events in the order in which they occur. 1.​Virus invades cell with CD4 antigen. 2.​Viral RNA converts with reverse transcriptase to viral DNA. 3.​Viral DNA integrates with host cell DNA. 4.​Virus remains latent, or actively replicates. 5.​Virus sheds protein coat.

Answer: 1, 3, 4, 5, 2 Explanation: The HIV virus gains entry into helper T cells, uses the cell DNA to replicate, interferes with normal function of the T cells, and destroys the normal cells.

What is the priority nursing diagnosis for a patient experiencing chemotherapy-induced anemia? a. Risk for injury related to poor blood clotting b. Fatigue related to decreased cellular oxygenation c. Disturbed body image related to skin color changes d. Imbalanced nutrition, less than body requirements related to anorexia

ANS: B Decreased numbers of red blood cells (RBCs) result in decreased cellular oxygenation and less energy. Decreased numbers of RBCs alone do not change the patient's blood-clotting ability. Although skin color changes and altered nutrition also occur with anemia, fatigue due to decreased cellular oxygenation is a priority nursing diagnosis.

List the pathophysiology concepts related to the onset of sepsis in sequential order. 1. Macrophage-producing cytokines are released. 2. Endotoxin released by microorganisms sets off an out-of-control inflammatory process. 3. Neutrophils arrive and multiply, occluding capillaries. 4. Vasodilation with increased capillary permeability and fluid leak.

Answer: 2, 1, 4, 3 Explanation: The pathophysiology of sepsis occurs as follows: Endotoxin released by microorganisms sets off an out-of-control inflammatory process; macrophage-producing cytokines are released; vasodilation with increased capillary permeability and fluid leak; neutrophils arrive and multiply, occluding capillaries.

A pediatric client is receiving chemotherapy for acute lymphocytic leukemia. The nurse recognizes that a potential oncological emergency for this client would be tumor lysis syndrome. For which manifestations should the nurse monitor this client? Select all that apply. A) Thrombocytopenia B) Altered levels of consciousness C) Respiratory distress D) Oliguria E) Upper-extremity edema

Answer: B, D Explanation: A) Tumor lysis causes a metabolic emergency. Because of electrolyte imbalance, the signs can be oliguria and altered levels of consciousness. Thrombocytopenia occurs with a hematological emergency. Space-occupying lesions can cause respiratory distress and upper-extremity edema.

A nurse in the Intensive Care Unit (ICU) is assigned a client diagnosed with AIDS. The type of precautions the nurse will adhere to in caring for this client is: A) Droplet. B) Reverse. C) Standard. D) Contact.

Answer: C Explanation: A) Healthcare workers can prevent most exposures to HIV by using standard precautions. With standard precautions, the healthcare professionals treat all clients alike, eliminating the need to know their HIV status. Treat all high-risk body fluids as if they are infectious, and use barrier precautions to prevent skin, mucous membrane, or percutaneous exposure to these fluids.

The nurse is discharging an HIV-positive pediatric client who has recently developed AIDS. The nurse is teaching the client's mother about health promotion activities for the child. It is important for the nurse to tell the mother that the client should not receive which immunizations due to HIV/AIDS status? A) Varicella vaccine B) Haemophilus influenzae type B (HIB conjugate vaccine) C) Hepatitis B vaccine (hep B) D) Diphtheria and tetanus toxoids and acellular pertussis vaccine (DTaP)

Answer: A Explanation: A) A child with an immune disorder such as HIV/AIDS should not be immunized with a live varicella vaccine, because of the risk of contracting the disease. DTaP, HIB, and hepatitis B vaccinations are not live vaccines, and should be given on schedule.

The mother of an Anna, an 8-year-old girl infected with HIV, is describing Anna's condition and activities to the nurse. Which statements would indicate positive outcomes for the child? Select all that apply. A) "Anna is attending school and doing well in her class." B) "Anna seems somewhat isolated and doesn't have any real friends." C) "Anna has a good appetite and eats regular meals." D) "Anna hasn't shown any sign of infection." E) "Anna attends a weekly support group for kids with HIV."

Answer: A, C, D, E Explanation: A) Positive outcomes for an HIV client would include remaining free from secondary infection, displaying normal nutritional patterns, demonstrating adequate coping with the stress of chronic disease, and attending school.

An 80-year-old client is recovering in the ICU from septicemia. Which intervention will help prevent further infection for this client? A) Foley drainage on the bed at the client's feet B) Oral and skin care C) Turn, cough, and deep breathe q shift. D) Sterile wound care

Answer: B Explanation: A) Good oral and skin care will prevent breakdown and prevent entry by bacteria. In order to prevent skin breakdown and promote respiratory function, the client is turned at least every 2 hours. The Foley drainage bag is always kept below the level of the client's bladder to prevent reflux. There is no evidence that this client has a wound.

The nurse is caring for a client with leukemia who is experiencing neutropenia as a result of chemotherapy. Which action should the nurse include in the plan of care for this client? A) Replace hand hygiene with gloves. B) Restrict visitors with communicable illnesses. C) Restrict fluid intake. D) Insert an indwelling urinary catheter to prevent skin breakdown.

Answer: B Explanation: A) In the neutropenic client, visitors with communicable infections should be restricted. Fluid intake should be encouraged. Gloves may be appropriate but should never replace hand hygiene. Invasive procedures such as indwelling catheters should be avoided.

The nurse is providing instructions to a client who has a prescription for a nonsteroidal anti-inflammatory drug (NSAID). What information is priority for the nurse to explain to the client about this medication? A) "Take your medication on an empty stomach." B) "Drink at least 8-10 glasses of water a day while taking your medication." C) "Constipation is common with your medication; include roughage in your diet." D) "Take your medication with food."

Answer: B Explanation: A) Nonsteroidal anti-inflammatory drugs (NSAIDS) are nephrotoxic; keeping the client well hydrated will help prevent kidney damage. Taking the medication with food will decrease gastrointestinal (GI) irritation, but preventing kidney damage is more of a priority. Taking the medication on an empty stomach will increase gastrointestinal (GI) irritation. Constipation is not an issue with NSAIDs.

The nurse is caring for a client who has been admitted to the unit with tuberculosis. The client is placed in isolation. To protect the caregivers and other clients on the unit, which type of isolation room is most appropriate? A) Single-door room with positive air flow (Air flows out of the room.) B) Isolation room with an anteroom and negative air flow (Air flows into the room.) C) Isolation room with an anteroom and normal airflow D) Single-door room with normal airflow

Answer: B Explanation: A) Patients with airborne infections such as meningococcemia, SARS, or TB are placed in an isolation room with an anteroom and negative pressure airflow. Air flows into the room and is vented in a special manner to prevent the organism from entering the rest of the unit. Positive flow rooms are used for those clients who are immunosuppressed so that microorganisms from the unit are not drawn into the room. Single-door isolation with normal airflow might be used for a client with droplet or wound infection. Single-door rooms are not equipped to have positive or negative airflow.

A nurse working in the pediatric intensive care unit (PICU) is caring for a pediatric client with HIV. The client is severely symptomatic with the additional diagnoses of lymphoma and wasting syndrome. The nurse understands that the client is in which clinical stage of HIV? A) Category N B) Category C C) Category A D) Category B

Answer: B Explanation: A) The 1994 Revised HIV Pediatric Classification System remains the standard for determining clinical staging and related treatment for children with HIV. The client described is Category C, a severely symptomatic client with lymphoma and wasting syndrome. The other choices are incorrect.

A pediatric client being treated for acute lymphocytic leukemia (ALL) has a white blood cell count of 1,000/mm3. Which nursing diagnosis would be a priority for this client? A) Readiness for Enhanced Immunization Status B) Impaired Gas Exchange C) Risk for Infection D) Activity Intolerance

Answer: C Explanation: A) In leukemia, the WBCs that are present are immature and incapable of fighting infection. The client with a WBC count of 500-1,000/mm3 is considered a moderate risk for infection. The client may or may not have activity intolerance, but it is not the priority nursing diagnosis. Impaired gas exchange is not evident in this client. Children with cancer would not be receiving immunizations during treatment.

The nurse is planning care for a client with multiple sclerosis. Which intervention would address the nursing diagnosis of Fatigue? A) Encourage increased activity. B) Schedule physical therapy three times a day. C) Plan activities with sufficient rest periods. D) Group activities together so care will not be interrupted.

Answer: C Explanation: A) Interventions to address the client's diagnosis of Fatigue include assessing the level of fatigue, arranging activities to include rest periods, and assisting the client to set priorities regarding activities. Activities should not be grouped together. Increased activity will not help the client with fatigue. Physical therapy three times a day may be too aggressive for this client.

A young adult client complains of blurred vision and muscle spasms that come and go over the past several months. On what information from the client's history should the nurse focus to help identify this help problem? A) Family history of Parkinson disease B) Family history of epilepsy C) Is an immigrant from Germany D) Has been depressed

Answer: C Explanation: A) Multiple sclerosis is primarily a disease of people of northern European ancestry. The onset of multiple sclerosis is usually between the ages of 20 and 50, with the peak at age 30. Family history of epilepsy, Parkinson disease, and depression are important items of the client's history but do not support a diagnosis of MS.

The nurse is preparing to assess a 90-year-old client admitted with tuberculosis. Which manifestation will the nurse most likely assess in this client? A) Night sweats B) Swollen lymph nodes C) Cough D) Hemoptysis

Answer: C Explanation: A) Presenting symptoms of tuberculosis in the older adult are often vague and include coughing, weight loss, diminished appetite, and periodic fevers. Night sweats, swollen lymph nodes, and hemoptysis are not considered presenting symptoms of tuberculosis in the older adult.

During an assessment, the nurse notes that a client receiving radiation treatments for breast cancer has excoriated skin. What is the priority nursing diagnosis for this client? A) Excess Fluid Volume B) Ineffective Breathing Pattern C) Risk for Infection D) Activity Intolerance

Answer: C Explanation: A) Radiation causes skin excoriation. With the excoriation, the client is at risk for infection due to skin breakdown. The client who receives radiation is more at risk for fluid volume deficit. Depending on the assessment, the client may or may not have activity intolerance. There is no evidence of respiratory difficulties in this client.

The nurse is preparing an educational session on sepsis. What should the nurse include as a major risk factor for the development of this health problem? A) Pneumococcal bacteria B) Leukocytosis on the complete blood count C) Undiagnosed urinary tract infection D) Elevated temperature

Answer: C Explanation: A) Sepsis is an entire-body inflammatory process. Sepsis is most often the result of gram-positive infections from Staphylococcus and Streptococcus bacteria but may also follow gram-negative bacterial infections such as Pseudomonas, Escherichia coli, and Klebsiella. A portal of entry for sepsis is the urinary system. Leukocytosis occurs with sepsis if the client is able to mount an immune response. An elevated temperature is a manifestation of sepsis.

A client being treated for cancer has a tumor designation of Stage IV, T4, N3, M1. What does this staging indicate to the nurse? A) The tumor will respond to chemotherapy. B) The tumor is small in size. C) The tumor has metastasized with lymph node involvement. D) There is one single tumor to treat.

Answer: C Explanation: A) T refers to the depth of invasion. N refers to the absence or presence and extent of lymph node involvement. M refers to presence of metastasis. The numbers range from 0 to 4, with higher numbers indicating increased size and metastasis. Stage IV indicates metastasis. The staging system is not used to determine tumor response to chemotherapy.

A client is admitted to the Intensive Care Unit with a systemic infection. What manifestations will the nurse most likely assess in this client? Select all that apply. A) Tachycardia B) Pain C) Edema D) Anorexia E) Fever Answer: A, D, E

Explanation: A) Fever, tachycardia, and anorexia are the most common symptoms of a systemic infection. Edema and pain are symptoms of a local infection.

HIV infects and destroys CD4 cells. List the following events in the order in which they occur. 1.​Virus invades cell with CD4 antigen. 2.​Viral RNA converts with reverse transcriptase to viral DNA. 3.​Viral DNA integrates with host cell DNA. 4.​Virus remains latent, or actively replicates. 5.​Virus sheds protein coat.

Answer: 1, 3, 4, 5, 2 Explanation: The HIV virus gains entry into helper T cells, uses the cell DNA to replicate, interferes with normal function of the T cells, and destroys the normal cells.

The nurse is caring for a patient who received a bone marrow transplant 10 days ago. The nurse would monitor for which of the following clinical manifestations that could indicate a potentially life-threatening situation? a. Mucositis b. Confusion c. Depression d. Mild temperature elevation

ANS: D During the first 100 days after a bone marrow transplant, patients are at high risk for life-threatening infections. The earliest sign of infection in an immunosuppressed patient can be a mild fever. Mucositis, confusion, and depression are possible clinical manifestations but are representative of less life-threatening complications.

In order to provide the best intervention for a patient, the nurse is often responsible for obtaining a sample of exudate for culture. What information will this provide? a. Whether a patient has an infection. b. Where an infection is located. c. What cells are being utilized by the body to attack an infection. d. What specific type of pathogen is causing an infection.

ANS: D People can transmit pathogens even if they don't currently feel ill. Some carriers never experience the full symptoms of a pathogen. A CBC will identify that the patient has an infection. Inspection and radiography will help identify where an infection is located. The CBC with differential will identify the white blood cells being used by the body to fight an infection. The culture will grow the microorganisms in the sample for identification of the specific type of pathogen.

In caring for a patient following lobectomy for lung cancer, which of the following should the nurse include in the plan of care? a. Position the patient on the operative side only. b. Avoid administering narcotic pain medications. c. Keep the patient on strict bed rest. d. Instruct the patient to cough and deep breathe.

ANS: D Postoperative deep breathing and coughing is important to promote oxygenation and clearing of secretions. Pain medications will be given to lessen pain and allow for deep breathing and coughing. Strict bed rest is not instituted, because early ambulation will help lessen postoperative complications such as deep vein thrombosis. Prolonged lying on the operative side is avoided.

The nurse is caring for a patient who was started on intravenous antibiotic therapy earlier in the shift. As the second dose is being infused, the patient reports feeling dizzy and having difficulty breathing and talking. The nurse notes that the patient's respirations are 26 breaths/min with a weak pulse of 112 beats/min. The nurse suspects that the patient is experiencing which condition? a. Suppressed immune response b. Hyperimmune response c. Allergic reaction d. Anaphylactic reaction

ANS: D The patient is exhibiting signs and symptoms of an anaphylactic reaction to the medication. These signs and symptoms during administration of a medication do not correspond to a suppressed immune response but a type of hyperimmune response. While the patient is experiencing a hyperimmune response, the signs and symptoms allow for a more specific response. While the patient is experiencing an allergic reaction, the signs and symptoms presented in the scenario allow for a more specific response. REF: Page 221 - Page 225

The nurse in an inner city clinic is providing a health screening for a homeless man with a history of drug abuse. The client has a chronic non-productive cough. For what should the nurse expect to screen this client? A) Herpes zoster B) Sickle cell disease C) Sick sinus syndrome D) Tuberculosis

Answer: D Explanation: A) The homeless client who abuses drugs is at risk for contracting tuberculosis. The nurse would expect to screen this client for TB. There is no evidence to support the need to screen the client for sickle cell disease, herpes zoster, or sick sinus syndrome.

A female patient complains of a "scab that just won't heal" under her left breast. During your conversation, she also mentions chronic fatigue, loss of appetite, and slight cough, attributed to allergies. What is the nurse's best action? a. Continue to conduct a symptom analysis to better understand the patient's symptoms and concerns. b. End the appointment and tell the patient to use skin protection during sun exposure. c. Suggest further testing with a cancer specialist and provide the appropriate literature. d. Tell her to put a bandage on the scab and set a follow-up appointment in one week.

ANS: A A comprehensive health history is vital to treating and caring for the patient. Often times, symptoms are vague. The nurse should conduct a symptom analysis to gather as much information as possible. Questions should address the duration of the symptoms and include the location, characteristics, aggravating and relief factors, and any treatments taken thus far.

A patient is not certain whether she and her family should participate in a genetic screening plan. She asks the nurse why the X-linked recessive disorder that has been noted in some of her family members is expressed in males more frequently than in females. What is the nurse's best response? a. "The disease tends to show up in males because they do not have a second X chromosome to balance the expression of the gene." b. "One X chromosome of a pair is always inactive in females. This inactivity effectively negates the effects of the gene." c. "Females are known to have more effective DNA repair mechanisms than males, thus negating the damage caused by the recessive gene." d. "Expression of genes from the male's Y chromosome does not occur in females, so they are essentially immune to the effects of the gene."

ANS: A Because the number of X chromosomes in males and females is not the same (1:2), the number of X-linked chromosome genes in the two genders is also unequal. Males have only one X chromosome for any gene on the X chromosome. As a result, X-linked recessive genes have a dominant expressive pattern of inheritance in males and a recessive expressive pattern of inheritance in females. This difference in expression occurs because males do not have a second X chromosome to balance the expression of any recessive gene on the first X chromosome. It is incorrect to say that one X chromosome of a pair is always inactive in females or that females have more effective DNA repair mechanisms than males. Also, it is not true that females can be immune to the effects of a gene, because genes from the male's Y chromosome are not expressed in females.

While collecting a health history on a patient admitted for colon cancer, which of the following questions would be a priority to ask this patient? a. "Have you noticed any blood in your stool?" b. "Have you been experiencing nausea?" c. "Do you have back pain?" d. "Have you noticed any swelling in your abdomen?"

ANS: A Early colon cancer is often asymptomatic, with occult or frank blood in the stool being an assessment finding in a patient diagnosed with colon cancer. If pain is present, it is usually lower abdominal cramping. Constipation and diarrhea are more frequent findings than nausea or ascites.

On admission to the clinic, the nurse notes a moderate amount of serous exudate leaking from the patient's wound. The nurse realizes what information about this fluid? a. Contains the materials used by the body in the initial inflammatory response. b. Indicates that the patient has an infection at the site of the wound. c. Is destroying healthy tissue. d. Results from ineffective cleansing of the wound area.

ANS: A Exudate is fluid moved from the vascular spaces to the area around a wound. It contains the proteins, fluid, and white blood cells (WBCs) needed to contain possible pathogens at the site of injury. Exudate appears as part of all inflammatory responses and does not mean an infection is present. Exudate is part of normal inflammatory responses which contain self-monitoring mechanisms to help prevent damage to healthy tissue. Exudate appears at wound sites regardless of cleaning done to the area of injury.

While planning care for a patient experiencing fatigue due to chemotherapy, which of the following is the most appropriate nursing intervention? a. Prioritization and administration of nursing care throughout the day b. Completing all nursing care in the morning so the patient can rest the remainder of the day c. Completing all nursing care in the evening when the patient is more rested d. Limiting visitors, thus promoting the maximal amount of hours for sleep

ANS: A Pacing activities throughout the day conserves energy, and nursing care should be paced as well. Fatigue is a common side effect of cancer and treatment; and while adequate sleep is important, an increase in the number of hours slept will not resolve the fatigue. Restriction of visitors does not promote healthy coping and can result in feelings of isolation.

An 18-month-old female patient is diagnosed with her fifth ear infection in the past 10 months. The physician notes that the child's growth rate has decreased from the 60th percentile for height and weight to the 15th percentile over that same time period. The child has been treated for thrush consistently since the third ear infection. The nurse understands that the patient is at risk for which condition? a. Primary immunodeficiency b. Secondary immunodeficiency c. Cancer d. Autoimmunity

ANS: A Primary immunodeficiency is a risk for patients with two or more of the listed problems. Secondary immunodeficiency is induced by illness or treatment. Cancer is caused by abnormal cells that will trigger an immune response. Autoimmune diseases are caused by hyperimmunity. REF: Page 221

The nurse is caring for a postoperative patient who had an open appendectomy. The nurse understands that this patient should have some erythema and edema at the incision site 12 to 24 hours post operation dependent on which condition? a. His immune system is functioning properly. b. He is properly vaccinated. c. He has an infection. d. The suppressor T-cells in his body are activated.

ANS: A Tissue integrity is closely associated with immunity. Openings in the integumentary system allow for the entrance of pathogens. If the immune response is functioning optimally, the body responds to the insult to the tissue by protecting the area from invasion of microorganisms and pathogens with inflammation. Routine vaccinations have no bearing on the body's response to intentional tissue impairment. The redness and swelling at the incision site in the first 12 to 24 hours is part of optimal immune functioning. A patient with erythema and edema that persist or worsen should be evaluated for infection. Suppressor T-cells help to control the immune response in the body. REF: Page 225

Individuals of low socioeconomic status are at an increased risk for infection because of which of the following? (Select all that apply.) a. Uninsured or underinsured status b. Easy access to health screenings c. High cost of medications d. Inadequate nutrition e. Mostly female gender

ANS: A, C, D Individuals of low socioeconomic status tend to be part of the underinsured or uninsured population. Lack of insurance decreases accessibility to health care in general and health screening services specifically. High costs of medication and nutritious food also make this population at higher risk for infection. Gender has not been shown to be an increased risk factor for infection in the lower socioeconomic population.

The nurse assesses the patient and notes all of the following. Select all of the findings that indicate the systemic manifestations of inflammation. a. Oral temperature 38.6° C/101.5° F b. Thick, green nasal discharge c. Patient complaint of pain at 6 on a 0 to 10 scale on palpation of frontal and maxillary sinuses d. WBC 20 cells/McL × 109/L e. Patient reports, "I'm tired all the time. I haven't felt like myself in days."

ANS: A, D, E Systemic manifestations of inflammatory response include elevated temperature, leukocytosis, and malaise and fatigue. Purulent exudates and pain are both considered local manifestations of inflammation.

A patient with prostate cancer is taking estrogen daily to control tumor growth. He reports that his left calf is swollen and painful. Which of the following would be the nurse's best action? a. Instruct the client to keep the leg elevated. b. Measure the calf circumference and compare the measurement with the right calf circumference measurement. c. Apply ice to the calf after a 10-minute massage of the area. d. Document assessment findings as an expected response with estrogen therapy.

ANS: B An adverse reaction to hormonal manipulation therapy is the development of thrombus formation. Massaging a calf that is swollen and painful is never correct, because this action might break a clot, causing formation of an embolus, which could then travel to the lungs.

A patient is being treated with an antibiotic for an infected orthopedic injury. What explanation should the nurse give to the patient about this medication? a. "Antibiotics will decrease the pain at the site." b. "An antibiotic helps to kill the infection causing the inflammation." c. "An antibiotic inhibits cyclooxygenase, an enzyme in the body." d. "Antibiotics will reduce the patient's fever."

ANS: B Antimicrobials treat the underlying cause of the infection which leads to inflammation. Analgesics and nonsteroidal antiinflammatory drugs (NSAIDs) help to treat pain. NSAIDs and other antipyretics are cyclooxygenase inhibitors. Antipyretics help to reduce fever.

A patient comes to the clinic with a complaint of painful, itchy feet. On interview, the patient tells the nurse that he is a college student living in a dormitory apartment that he shares with five other students. What teaching should the nurse provide for this patient? a. "Don't eat with the other students." b. "Avoid sharing razors and other personal items." c. "Have a complete blood count (CBC) checked monthly." d. "Disinfect showers and bathroom floors weekly after use."

ANS: B Avoidance of sharing personal items like razors and hairbrushes can decrease the spread of pathogens that cause inflammation and infection. Not eating with the others in his college apartment won't relieve or prevent the spread of infection. A CBC monthly will not treat or prevent inflammation. Showers should be disinfected before and after each use.

A patient is diagnosed with a sprain to her right ankle after a fall. The patient asks the nurse about using ice on her injured ankle. What is the nurse's best response? a. "Use ice only when the ankle hurts." b. "Ice should be applied for 15 to 20 minutes every 2 to 3 hours over the next 1 to 2 days." c. "Wrap an ice pack around the injured ankle for the next 24 to 48 hours." d. "Ice is not recommended for use on the sprain because it would inhibit the inflammatory response."

ANS: B Ice is used on areas of injury during the first 24 to 48 hours after the injury occurs to prevent damage to surrounding tissues from excessive inflammation. Ice should be used for a maximum of 20 minutes at a time every 2 to 3 hours. Ice must be used according to a schedule for it to be effective and not be overused. Using ice more often or for longer periods of time can cause additional tissue damage. Ice is recommended to inhibit the inflammatory process from damaging surrounding tissue.

A cancer patient's susceptibility to the syndrome of inappropriate antidiuretic hormone (SIADH) can be suspected with which of the following laboratory results? a. Serum potassium of 5.2 mmol/L b. Serum sodium of 120 mmol/L c. Hematocrit of 40% d. Blood urea nitrogen (BUN) of 10 mg/dL

ANS: B In SIADH, there is increased secretion of antidiuretic hormone (ADH) from the posterior pituitary gland, leading to increased water reabsorption from the distal convoluted tubule and collecting duct. As a result, weight increases and serum sodium and hematocrit levels are diluted, and, hence, a low serum sodium level is seen. ADH has no direct effect on serum potassium, because potassium is primarily an intracellular electrolyte and changes in the serum concentration of potassium rarely occur in SIADH. The blood urea nitrogen is normal.

The nurse would incorporate which of the following into the plan of care as a primary prevention strategy for reduction of the risk for cancer? a. Yearly mammography for women aged 40 years and older b. Using skin protection during sun exposure while at the beach c. Colonoscopy at age 50 and every 10 years as follow-up d. Yearly prostate specific antigen (PSA) and digital rectal exam for men aged 50 and over

ANS: B Primary prevention of cancer involves avoidance to known causes of cancer, such as sun exposure. Secondary screening involves physical and diagnostic examination.

Which set of assessment data is consistent for a patient with severe infection that could lead to system failure? a. Blood pressure (BP) 92/52, pulse (P) 56 beats/min, respiratory rate (RR) 10 breaths/min, urine output 1200 mL in past 24 hours b. BP 90/48, P 112 beats/min, RR 26 breaths/min, urine output 240 mL in past 24 hours c. BP 112/64, P 98 beats/min, RR 18 breaths/min, urine output 2400 mL in past 24 hours d. BP 152/90, P 52 beats/min, RR 12 breaths/min, urine output 4800 mL in past 24 hours

ANS: B The patient with severe infection presents with low BP and compensating elevations in pulse to move lower volumes of blood more rapidly and respiration to increase access to oxygen. Urine output decreases to counteract the decreased circulating blood volume and hypotension. These vital signs are all too low: Blood pressure (BP) 92/52, pulse (P) 56 beats/min, respiratory rate (RR) 10 breaths/min, urine output 1200 mL in past 24 hours. The patient with severe infection does have a low BP, but the pulse and respiratory rate increase to compensate. This data is all within normal limits: BP 112/64, P 98 beats/min, RR 18 breaths/min, urine output 2400 mL in past 24 hours. This set of data reflects an elevated BP with a decrease in pulse and respiratory rates along with normal urine output: BP 152/90, P 52 beats/min, RR 12 breaths/min, urine output 4800 mL in past 24 hours. None of these is a typical response to severe infection.

The nurse is caring for a patient who is being discharged home after a splenectomy. What information on immune function needs to be included in this patient's discharge planning? a. The mechanisms of the inflammatory response b. Basic infection control techniques c. The importance of wearing a face mask in public d. Limiting contact with the general population

ANS: B The spleen is one of the major organs of the immune system. Without the spleen, the patient is at higher risk for infection; so, the nurse must be sure that the patient understands basic principles of infection control. The patient with a splenectomy does not need to understand the mechanisms of inflammatory response. The patient with a splenectomy does not need to wear a face mask in public as long as the patient understands and maintains the basic principles of infection control. The patient who has had a splenectomy does not need to limit contact with the general population as long as the patient understands and maintains the basic principles of infection control. REF: Page 219

The nurse is caring for a patient with a diagnosed case of Clostridium difficile. The nurse expects to implement which of the following interventions? (Select all that apply.) a. Administration of protease inhibitors b. Use of personal protective equipment c. Patient teaching on methods to inhibit transmission d. Preventing visitors from entering the room e. Administration of intravenous fluids f. Strict monitoring of intake and output

ANS: B, C, E, F Protease inhibitors are used for treatment of viral infections, not bacterial infections. The nurse wants to protect visitors from exposure to the bacteria and protect the patient from secondary infection while immunocompromised, but the patient will need the support of family and close friends. Contact isolation precautions must be strictly followed along with the use of personal protective equipment and teaching on methods to inhibit transmission to help break the chain of infection. Intravenous fluids and strict intake and output monitoring will be important for the patient suffering the effects of Clostridium difficile, because it causes diarrhea with fluid loss.

The parents of a newborn question the nurse about the need for vaccinations: "Why does our baby need all those shots? He's so small, and they have to cause him pain." The nurse can explain to the parents that which of the following are true about vaccinations? (Select all that apply.) a. Are only required for infants b. Are part of primary prevention for system disorders c. Prevent the child from getting childhood diseases d. Help protect individuals and communities e. Are risk free f. Are recommended by the Centers for Disease Control and Prevention (CDC)

ANS: B, D, F Immunizations are considered part of primary prevention, help protect individuals from contracting specific diseases and from spreading them to the community at large, and are recommended by the CDC. Immunizations are recommended for people at various ages from infants to older adults. Vaccination does not guarantee that the recipient won't get the disease, but it decreases the potential to contract the illness. No medication is risk free. REF: Page 224

The nurse is working on a plan of care with her patient which includes turning and positioning and adequate nutrition to help the patient maintain intact skin integrity. The nurse helps the patient to realize that this breaks the chain of infection by eliminating which element? a. Host b. Mode of transmission c. Portal of entry d. Reservoir

ANS: C Broken or impaired skin creates a portal of entry for pathogens. By maintaining intact tissue, the patient and the nurse have broken the chain of infection by eliminating a portal of entry. Host is incorrect because you are not eliminating the person or organism. Intact tissue does not eliminate the mode of transmission. Skin can still be used to transfer pathogens regardless of it being intact or broken. Intact skin does not eliminate the location for pathogens to live and grow.

The nurse reviews the patient's complete blood count (CBC) results and notes that the neutrophil levels are elevated, but monocytes are still within normal limits. This indicates what type of inflammatory response? a. Chronic b. Resolved c. Early stage acute d. Late stage acute

ANS: C Elevated neutrophils and monocytes within normal limits are findings indicative of early inflammatory response. Neutrophils increase in just a few hours, while it takes the body days to increase the monocyte levels. Chronic inflammation results in varying elevations in WBCs dependent on multiple issues. Elevated neutrophils are not indicative of resolved inflammation. Elevations in monocytes occur later in the inflammatory response.

A nurse is instructing her patient with ulcerative colitis regarding the need to avoid enteric coated medications. The nurse knows that the patient understands the reason for this teaching when he states which of the following? a. "The coating on these medications is irritating to my intestines." b. " I need a more immediate response from my medications than can be obtained from enteric coated medications." c. "Enteric coated medications are absorbed lower in the digestive tract and can be irritating to my intestines or inadequately absorbed by my inflamed tissue." d. " I don't need to use these medications because they cause diarrhea, and I have had enough trouble with diarrhea and rectal bleeding over the past weeks."

ANS: C Enteric coatings on medications are designed to prevent breakdown and absorption of the medication until lower in the digestive tract, usually to prevent stomach irritation or to reach a certain point in the digestive tract for optimal absorption. For the patient with ulcerative colitis, the intestinal lining is inflamed or susceptible to inflammation and can have impaired absorption; therefore, enteric coated medications should be avoided. The coating is not irritating, but the medication can be. The response time of the medication is not a concern in this instance. Enteric coated medicines do not cause diarrhea simply because they are enteric coated.

While caring for a patient preparing for a kidney transplant, the nurse knows that the patient understands teaching on immunosuppression when she makes which statement? a. "My body will treat the new kidney like my original kidney." b. "I will have to make sure that I avoid being around people." c. "The medications that I take will help prevent my body from attacking my new kidney." d. "My body will only have a problem with my new kidney if the donor is not directly related to me."

ANS: C Immunosuppressant therapy is initiated to inhibit optimal immune response. This is necessary in the case of transplantation, because the normal immune response would cause the body to recognize the new tissue as foreign and attack it. The body will identify the new kidney as foreign and will not treat it as the original kidney. While patients with transplants must be careful about exposure to others, especially those who are or might be ill, and practice adequate and consistent infection control techniques, they don't have to avoid people or social interaction. The new kidney brings foreign cells regardless of relationship between donor and recipient. REF: Page 222

While reviewing the complete blood count (CBC) of a patient on her unit, the nurse notes elevated basophil and eosinophil readings. The nurse realizes that this is most indicative of which type of infection? a. Bacterial b. Fungal c. Parasitic d. Viral

ANS: C Parasitic infections are frequently indicated on a CBC by elevated basophil and eosinophil levels. Bacterial infections do not lead to elevated basophil and eosinophil levels but elevated B and T lymphocytes, neutrophils, and monocytes. Fungal infections do not lead to elevated basophil and eosinophil levels. Viral infections create elevations in B and T lymphocytes, neutrophils, and monocytes.

A nurse is teaching a group of business people about disease transmission. He knows that he needs to reeducate when one of the participants states which of the following? a. "When traveling outside of the country, I need to be sure that I receive appropriate vaccinations." b. "Food and water supplies in foreign countries can contain microorganisms to which my body is not accustomed and has no resistance." c. "If I don't feel sick, then I don't have to worry about transmitted diseases." d. "I need to be sure to have good hygiene practices when traveling in crowded planes and trains."

ANS: C People can transmit pathogens even if they don't currently feel ill. Some carriers never experience the full symptoms of a pathogen. Travelers may need different vaccinations when traveling to countries outside their own because of variations in prevalent microorganisms. Food and water supplies in foreign countries can contain microorganisms that will affect a body unaccustomed to their presence. Adequate hygiene is essential when in crowded, public spaces like planes and other forms of public transportation.

A client with a history of relapsing-remitting multiple sclerosis is expecting her first child. What would be indicated for this client? A) Suggest reproductive counseling, as multiple sclerosis can be genetic. B) Instruct to expect a period of remission after delivery of the baby. C) Instruct to expect an exacerbation of symptoms while pregnant. D) Discuss pain control during labor, as contractions will be severe.

Answer: A Explanation: A) A definite genetic factor has not been established; however, studies suggest that genetic factors make some individuals more susceptible to the disorder than others. Reproductive counseling would be recommended for this client. Pregnancy often brings about remission of multiple sclerosis, but with a slightly increased relapse rate postpartum. The strength of uterine contractions in a client with multiple sclerosis is not severe, and because clients often have lessened sensation, labor may be almost painless.

What could the nurse do to decrease the inflammation? A) Anti-inflammatory medication B) Diuretics C) Opioid medication D) Antibiotics

Answer: A Explanation: A) Anti-inflammatory medication will reduce the pain and inflammation caused by arthritis. Opioid medication is not usually indicated to treat an inflammatory process. Antibiotics would be ordered for an infection. Diuretics are not used to treat the inflammatory process.

A client with multiple sclerosis is prescribed diazepam (Valium). What assessment finding indicates that the medication is effective for the client? A) Muscle spasticity is reduced. B) Blood glucose level is within normal limits. C) The client states that muscles are weak. D) Ophthalmologic examination shows no evidence of cataracts.

Answer: A Explanation: A) Diazepam (Valium) is a muscle relaxant commonly used for clients with multiple sclerosis. Diazepam (Valium) does not cause muscle weakness. Evidence of medication effectiveness would be an observed reduction in muscle spasticity. Glucose intolerance would be assessed if the client were prescribed an adrenal corticosteroid. Cataract development is also a side effect of adrenal corticosteroids.

The nurse instructs an older client with arthritis on the side effects of nonsteroidal anti-inflammatory drug (NSAID) therapy. Which client statement would indicate that teaching had been effective? A) "I will Report any abnormal bruising." B) "Caffeine will decrease the effectiveness of the medication." C) "I cannot take other medications." D) "If I have a change in my mood I will call the prescriber."

Answer: A Explanation: A) Elderly clients are at risk for increased bleeding with nonsteroidal anti-inflammatory drug (NSAID) therapy. The client should be taught to report any abnormal bruising, which may indicate bleeding. Elderly clients often take several medications, and refraining from taking them with NSAIDs is an unrealistic outcome. Mood changes are not a side effect of NSAID therapy. There is no reason for avoiding use of caffeine while using an NSAID.

The nurse instructs a client recovering from a mastectomy on ways to prevent lymphedema. Which client statement indicates that teaching has been successful? A) "I should do the exercises on my affected arm every day." B) "I have to take no special precautions." C) "I should avoid cleansing my skin with soap." D) "Eating fresh fruits and vegetables will prevent my arm from swelling."

Answer: A Explanation: A) Range-of-motion exercises in the affected arm helps develop collateral drainage and prevent the development of lymphedema. The client should be instructed to protect the affected limb by not permitting blood pressure measurement and avoiding tight jewelry and clothing on the limb. There is no reason for the client to avoid cleansing the skin of the affected arm with soap. Consuming fresh fruits and vegetables will not prevent the development of lymphedema.

A client admitted with an exacerbation of multiple sclerosis is demonstrating frustration with eating because hand and arm spasms prevent the proper use of utensils. What should the nurse do to assist this client? A) Consult with Occupational Therapy regarding assistive devices for meals. B) Counsel the client to select finger foods for meals. C) Plan time to feed the client. D) Consult with Physical Therapy regarding hand and arm exercises.

Answer: A Explanation: A) Since the ability to feed oneself is essential to positive self-concept and self-esteem, the nurse should consult with Occupational Therapy for devices that the client can use to maintain independence at meal times. The nurse should not counsel the client to select finger foods for meals, or feed the client. This would not support the client's self-concept and self-esteem needs. Physical Therapy might be consulted for hand splints, but hand and arm exercises might not be beneficial for this client.

A client with relapsing-remitting multiple sclerosis tells the nurse that even though the primary symptoms of exacerbation are leg spasms and blurred vision, the hardest part is trying to get through the day because of being so tired. Which diagnosis should the nurse identify as a priority for this client? A) Fatigue B) Disturbed Sensory Perception C) Impaired Physical Mobility D) Self-Care Deficit

Answer: A Explanation: A) The client states that the worst part of the disease exacerbation is being tired even though leg spasms and blurred vision are present. The nurse should identify the diagnosis of Fatigue as being a priority for this client. The diagnoses of Impaired Physical Mobility because of the leg spasms and Disturbed Sensory Perception because of the blurred vision are additional nursing diagnoses applicable for this client, but they are not the priority based on the client's statement. The client may or may not have a Self-Care Deficit.

The nurse is reviewing the plan of care for a client being treated with brachytherapy for breast cancer. Which assessment finding indicates that the client's skin integrity has been maintained? A) Skin intact B) Skin dry and excoriated C) Skin stretched D) Skin damp and sweaty

Answer: A Explanation: A) The goal for the client receiving radiation therapy to the chest is intact skin, which the nurse would expect to find. Skin that is damp with sweat, dry, or stretched is not consistent with radiation. If the goal were not met, the nurse would find excoriation.

The nurse is providing discharge instructions to a client being treated for cancer. For which symptoms should the client be instructed to call for help at home? Select all that apply. A) Difficulty breathing B) Significant increase in vomiting C) Desire to end life D) Improved sense of well-being E) New onset of bleeding

Answer: A, B, C, E Explanation: A) The client should be instructed to call for help with any difficulty breathing, significant increase in vomiting, a desire to end life, or a new onset of bleeding. An increased sense of well-being would be a desired effect of treatment for cancer.

A client with multiple sclerosis is observed transferring from the bed to a motorized wheelchair and applying splints to the lower extremities before entering the bathroom to perform morning self-care. What could the nurse conclude regarding this observation? A) The client uses assistive devices to optimize autonomy. B) The client needs instruction to conduct morning care before applying splints to lower extremities. C) The client is dependent upon assistive devices. D) The client is reliant upon assistive devices for independent.

Answer: A Explanation: A) The nurse observed the client independently transfer from the bed to a motorized wheelchair, apply splints, and enter the bathroom to perform morning self-care. This is evidence that the client uses assistive devices to optimize autonomy. The statement "Client is reliant upon assistive devices for independence" indicates that the client is not autonomous. The statement "Client is dependent upon assistive devices" also indicates the client is not autonomous. The statement "Client needs instruction to conduct morning care before applying splints to lower extremities" does not take into consideration the client's preference, which might be to apply the splints before doing self-care.

The nurse is caring for a client who is newly diagnosed with HIV. The client asks the nurse if there are ways to protect the client's sexual partner. Which client statement indicates the need for further instruction? A) "I know to use an oil-based lubricant to prevent spread of the disease to my partner." B) "I know I can't donate blood anymore, as I have HIV." C) "I will not share my toothbrush or razor with my partner." D) "I know I have to practice safe sex with my partner by using a latex condom."

Answer: A Explanation: A) The nurse should educate the client regarding the prevention of the spread of HIV. The client will need further education when he states that he will use an oil-based lubricant. The client should be educated to use latex condoms for oral, vaginal, or anal intercourse and to avoid natural or animal skin condoms, which allow passage of HIV. The client should use only water-based lubricants-not oil-based, such as petroleum jelly, which can result in condom damage. The client is correct in stating that it is not an acceptable practice to share toothbrushes or razors. The client is also correct in stating that blood donation is prohibited.

The nurse is planning care for a 90-year-old client who was recently diagnosed with tuberculosis. The client lives alone in an apartment and will continue treatment at home. Which nursing diagnosis is a priority for this client? A) Ineffective Therapeutic Regimen Management B) Deficient Knowledge C) Ineffective Breathing Pattern D) Risk for Injury

Answer: A Explanation: A) The treatment regimen for tuberculosis requires that the client take many medications, maintain nutrition, and be aware of potential side effects. Due to increased age and normal forgetfulness, this client is at risk for ineffective treatment in the home. The client may have a knowledge deficit but the priority is the treatment regimen. Since the client is being treated in the home, there is not much risk for ineffective breathing. The client is at risk for injury because of age, not TB.

The nurse is caring for a client with metastatic breast cancer receiving chemotherapy. Even though the prognosis is poor, the client tells the nurse that the plan is to do everything to survive. How should the nurse respond to this client? A) "You have a great attitude and I am here to support you through education to help you survive." B) "It is important to plan for your death, even though there is a chance you will survive." C) "You should face the reality of the situation. You do not have a good chance of survival." D) "I am going to speak with your family regarding your unrealistic expectations."

Answer: A Explanation: A) This client is in the earliest stages of cancer treatment, with removal of the primary tumor about to take place. The nurse's role is to support this client's optimism and help in fighting the disease by teaching about nutrition and other supportive actions the client can take to minimize complications of treatment. While the prognosis may be poor, the outcome is not absolute, and the client's wish to do whatever is necessary to survive should be supported. Emphasizing the low survival rate, encouraging the client to prepare for death, and talking with the family about the client's unrealistic expectations would not support the client's optimism.

A 15-year-old client is brought to the Emergency Department with fatigue, weight loss, a dry cough, and night sweats. The family just recently immigrated to the United States. For which potential risk should the nurse plan this client's care? A) Pneumothorax B) Pneumonia C) Renal failure D) Septicemia

Answer: A Explanation: A) This client was foreign-born, a risk factor for TB, and has the classic symptoms of tuberculosis. The nurse plans frequent respiratory assessments, as this child is at risk for pneumothorax. Patients with TB are not at particular risk for pneumonia, renal failure, or septicemia.

A nurse is volunteering in a health screening booth at the state fair. The nurse has assessed several clients and determines that which client demonstrates the decline in responsiveness of the immune system of an older adult? A) An 88-year-old client with pneumonia who has a temperature of 99.5°F B) A 70-year-old client who has swelling and redness around an abdominal incision from an open appendectomy C) A 58-year-old client who complains of redness and itching after developing a rash from contact with poison ivy D) A 56-year-old client who has 8 mm induration at the site of a PPD skin test 72 hours earlier

Answer: A Explanation: The client who has only a slight elevation in temperature in response to pneumonia is an example of a decline in the expected immune response. The other clients are demonstrating an expected immune response as evidenced by redness, swelling, and induration.

During an outpatient clinic follow-up appointment, a 46-year-old client with multiple sclerosis (MS) has lab tests completed. The results show elevated levels of aspartate aminotransferase (AST), serum glutamic-oxaloacetic transaminase (SGOT), alanine aminotransferase (ALT), serum glutamic-pyruvic transaminase (SGPT), and alkaline phosphatase (ALP). What is the priority concern for the nurse?? Select all that apply. A) Adverse response to Avonex B) Adverse response to Aubagio C) Flare-up due to demyelination D) Adverse response to bisacodyl E) Damage from viral infection

Answer: A, B Explanation: A) AST, SGOT, ALT, SGPT and ALP are liver enzymes that are monitored to detect adverse response to the medications Avonex and Aubagio. Flare-ups due to demyelination do not cause liver enzymes to increase. Bisacodyl is a stool softener used for constipation in MS and does not cause liver enzymes to elevate. Incomplete evidence links viral infection to the risk of MS. There is no reason to attribute elevated liver enzymes to viral exposure.

A client, who has been given a yellow fever vaccine before traveling to the Amazon Basin, asks the nurse to explain how the elements of the immune system will now provide protection. Which is the appropriate response by the nurse? Select all that apply. A) "Human macrophages engulf the weakened vaccine virus as if it is dangerous and antigens stimulate the immune system to attack it." B) "In the lymph nodes, part of the lymphoid system, the macrophages present yellow fever antigens to T cells and B cells." C) "A response from yellow fever-specific T cells is activated. B cells secrete yellow fever antibodies." D) "The body's immune system eats away at the protective sheath (myelin) that covers the nerves." E) "The initial weak infection is eliminated and the client is left with a supply of memory T and B cells for future protection against yellow fever."

Answer: A, B, C, E Explanation: Macrophages ingest antigens and signal helper T cells that antigens are present. Lymph nodes filter foreign products or antigens from the lymph system and house and support proliferation of lymphocytes and macrophages. Antibodies directly attack and destroy antigens either before or after antigens invade body cells. The immune system damaging the myelin is an autoimmune response in MS. Memory B cells and T cells remember how to identify the antigen and will reactivate at a future time if the same type of antigen is present.

The nurse conducts an evaluation after completing a training session for community members on ways to prevent the onset of nephritis. When evaluating the success of this session, what responses should the members provide as evidence that learning has been successful? Select all that apply. A) Practicing good hygiene B) Not smoking C) Maintaining a healthy body weight D) Limiting alcohol intake E) Controlling high blood pressure

Answer: A, B, C, E Explanation: A) The exact cause of nephritis is unknown, but preventing viral infections through practicing good hygiene habits, not smoking, maintaining a healthy body weight, and controlling high blood pressure reduces the risk of developing this disease. Limiting alcohol intake will not affect the development of nephritis.

The nurse is teaching a 34-year-old client with client who has a sister and mother with a history of breast cancer about early screening for the health problem. Which should the nurse include in this teaching? Select all that apply. A) Routine monthly breast self-examination B) Annual screening mammography C) Routine breast exams to begin after age 35 D) Clinical breast examination every 3 years E) Reporting of any changes in breast tissue to the health provider at the next routine visit

Answer: A, B, D Explanation: A) American Cancer Society guidelines for cancer screening include routine breast self-examination starting at age 20; prompt reporting of any change in breast tissue to healthcare provider; clinical breast examination every 3 years from ages 20 to 39, and yearly thereafter; and annual screening mammography starting at age 40, except in women at increased risk, who may have more frequent mammography or other tests such as breast ultrasound exams. Since this client's mother and sister both have a history of breast cancer, she would be eligible for annual mammography.

A nurse working in the ICU is receiving a client diagnosed with early septic shock from the Emergency Department. The nurse will recognize which symptoms associated with this condition? Select all that apply. A) Normal blood pressure B) Rapid and deep respirations C) Shallow respirations D) Warm and flushed skin E) Lethargic mental status F) Decreased urine output

Answer: A, B, D Explanation: A) Septic shock has an early phase and a late phase. Early-phase manifestations include normal blood pressure, rapid and deep respirations, and warm or flushed skin. The other manifestations occur in the late phase of shock.

A client who has been diagnosed with untreated HIV comes in complaining of fatigue and weight loss. What are some important elements of the physical exam for evaluating the client's AIDS status? Select all that apply. A) Assess the general appearance. B) Assess skin color, temperature, and moisture. C) Assess hair loss. D) Inspect the skin for evidence of rashes or lesions. E) Inspect the mouth for lesions.

Answer: A, B, D, E Explanation: Assess height, weight, and body type for apparent weight loss or wasting. Pallor may indicate bone marrow suppression with accompanying immunodeficiency. Skin lesions may be indicative of Kaposi sarcoma. Hair loss is not usually related to AIDS. Mouth lesions, which may have a "cottage cheese" appearance, may indicate candida.

A nurse is caring for a client with chronic myeloid leukemia (CML) who is neutropenic. To ensure safety for the client, the nurse will: Select all that apply. A) Place client in reverse isolation. B) Place patient in standard precaution isolation. C) Administer granulocyte colony-stimulating factor (G-CSF) as ordered. D) Administer neutrophil colony-stimulating factor (N-CSF) as ordered. E) Administer a prophylactic gram-negative antibiotic.

Answer: A, C Explanation: A) A client who is neutropenic has a decrease in the level of white blood cells (WBCs) and is susceptible to infection and/or disease. To ensure the safety of the client with neutropenia, the nurse will place the client in reverse isolation, administer granulocyte colony-stimulating factor (G-CSF) as ordered, and administer a broad-spectrum antibiotic as ordered. Standard precautions should be used for all clients and this does not ensure safety of the neutropenic client.

A nurse is planning care for a client with leukemia. The nurse chooses "Risk for Bleeding" as the nursing diagnosis. What interventions support this nursing diagnosis? Select all that apply. A) Educate client in use of soft toothbrush for oral care. B) Use non-electric razor when providing grooming for client. C) Limit parenteral injections. D) Apply pressure to arterial puncture sites for 5 minutes. E) Encourage client to deep breathe and huff cough frequently.

Answer: A, C Explanation: A) The client at risk for bleeding has specific interventions to which the nurse should adhere. The nurse should educate the client in the use of soft toothbrush and the use of an electric razor to avoid bleeding. The nurse should also limit the use of parenteral injections and apply 15-20 minutes of pressure to any arterial puncture sites. The nurse should discourage the client to forcefully cough to prevent further bleeding.

The nurse is teaching a class at a local community center about decreasing risk factors for cancer. Which risk factors should the nurse include in the teaching regarding leukemia? Select all that apply. A) Alkylating agents B) Diets low in fat C) Exposure to infectious agents D) Bloom syndrome E) Decreased exercise

Answer: A, C, D Explanation: A) A higher incidence of leukemia associated with chromosomal defects such as Bloom syndrome, exposure to infectious agents, and chemical agents used to treat previous cancer, such as alkylating agents. Low-fat diets are not a risk factor for leukemia, and neither is lack of exercise.

A nurse working with a 52-year-old woman who has been prescribed NSAIDs as part of her treatment for rheumatoid arthritis should assist the client by: Select all that apply. A) Monitoring for signs of allergic reaction. B) Assuring the client that there is no relationship between NSAIDs and heart disease. C) Encouraging the client to take with a full glass of water, milk, or small snack to help avoid GI distress. D) Monitoring for signs of renal problems. E) Advising against abrupt discontinuation of drugs.

Answer: A, C, D, E Explanation: An aspirin allergy or sensitivity, or a reaction to NSAIDs, can cause symptoms that range from mild to severe, especially in people with asthma. NSAIDs have been linked to heart failure. Taking NSAIDs with food may help reduce irritation of the stomach and prevent an ulcer. If you take NSAIDs in high doses, the reduced blood flow can permanently damage your kidneys and it can eventually lead to kidney failure and require dialysis. Abrupt discontinuation can have serious side effects.

The nurse instructs a group of community members on the difference between benign and malignant neoplasms. Which participant statements indicate that teaching has been effective? Select all that apply. A) "Benign tumors grow slowly." B) "Malignant tumors are easy to remove." C) "Benign tumors stay in one area." D) "Malignant tumors push other tissue out of the way." E) "Malignant tumors can grow back."

Answer: A, C, E Explanation: A) Benign tumors are slow-growing, stay in one area, are easy to remove, and push other tissue out of the way. Malignant tumors are more difficult to remove. They invade neighboring tissue and can return once removed.

The nurse is caring for a client newly admitted to the medical-surgical unit with glomerulonephritis. Which classic manifestations of this disorder should the nurse expect to assess in this client? Select all that apply. A) Edema B) Weight loss C) Hematuria D) Acute flank pain E) Proteinuria

Answer: A, C, E Explanation: A) Hematuria, proteinuria, and edema are the classic signs of glomerular disease because this disorder affects the structure and function of the glomerulus, disrupting glomerular filtration. This increased permeability in the glomeruli leads to plasma proteins being lost in the urine as well as red blood cells. Edema occurs because of the loss of plasma proteins, namely albumin. Acute flank pain is often more characteristic of renal calculi. Weight loss is a nonspecific symptom that can occur with many disorders, but not typically with glomerulonephritis.

The nurse is reviewing data collected during a health history and physical assessment and determines that a client is at risk for developing breast cancer. What did the nurse most likely assess in this client? Select all that apply. A) Age 60 B) Breastfed both children C) Sister had breast cancer D) Body mass index 22 E) Menopause at age 58

Answer: A, C, E Explanation: A) The risk for developing breast cancer increases with age. Having a first-degree relative with breast cancer increases the risk. Menopause after the age of 55 also increases the risk for developing breast cancer. Breastfeeding and maintaining a normal body weight lower a person's risk for developing breast cancer.

A client with breast cancer is receiving 5-fluorouracil (5-FU). Based on knowledge of this medication and its anticipated adverse effects or side effects, which actions should the nurse perform? Select all that apply. A) Assess CBC results. B) Encourage daily fluid intake of 2-3 liters. C) Monitor ECG. D) Test stool for occult blood. E) Assess lung sounds.

Answer: A, D Explanation: A) The antimetabolite fluorouracil interferes with pyrimidine and purine synthesis, which are essential for DNA production. The nurses should monitor CBC with differential, electrolytes, and kidney and liver function studies; monitor for bleeding and protect client from traumatic injury; monitor for signs of infection; and monitor for dyspnea and cough. Encouraging a daily fluid intake of 2-3 liters is recommended for clients receiving alkylating agents because they could potentially develop renal failure. Monitoring the ECG is recommended in clients receiving antitumor antibiotics. Assessing lung sounds is recommended in clients receiving alkylating agents, due to the potential for developing pulmonary fibrosis.

The nurse caring for a homeless client at risk for tuberculosis will include which symptoms of the disease when educating the client? Select all that apply. A) Fatigue B) Low-grade morning fever C) Productive cough that later turns to a dry, hacking cough D) Weight loss E) Night sweats

Answer: A, D, E Explanation: A) Manifestations of tuberculosis often develop insidiously and are initially nonspecific. Fatigue, weight loss, diminished appetite, low-grade afternoon fever, and night sweats are common. A dry cough develops, which later becomes productive of purulent and/or blood-tinged sputum. It is often at this stage that the client first seeks medical attention.

The nurse is planning care for a client with acute myeloid leukemia (AML). Which diagnoses are priorities for this client to minimize the risk of complications associated with AML? Select all that apply. A) Risk for Infection B) Ineffective Thermoregulation C) Imbalanced Nutrition D) Fluid Volume Excess E) Risk for Ineffective Protection (Bleeding)

Answer: A, E Explanation: A) AML results in neutropenia (decreased neutrophils = risk of infection) and thrombocytopenia (decreased platelets, which leads to increased risk of bleeding). Therefore, actions to minimize these risks include caution when moving or assisting the client to move, as well as strict hand hygiene to prevent possible cross-contamination. Weight loss is a symptom of chronic myeloid leukemia (CML), not AML. Therefore, dietary needs are not increased with AML. Restriction of fluids and salt are not needed. The client with AML does not have a problem with fluid shifts or edema that would require these restrictions. Fluids are encouraged to remove wastes that occur with chemotherapy treatment and cellular breakdown. Heat intolerance is a symptom of CML, not AML. CML has heat intolerance due to hypermetabolism state present with the condition.

The nurse has completed a seminar teaching a group in the community about ways to reduce cancer risks. The nurse returns a month later to evaluate the effectiveness of the seminar. Which statements made by members of the group indicate retention and application of the material presented by the nurse to reduce the risk of developing cancer? Select all that apply. A) "I stopped using tanning booths." B) "I began drinking two glasses of red wine a day with dinner." C) "I have reduced my intake of fiber." D) "I have increased the amount of lean red meat in my diet." E) "I now limit my alcohol intake to three drinks per week."

Answer: A, E Explanation: A) Excessive use of alcohol, especially in women, has been linked to increased risk of breast cancer, so reduction in intake would demonstrate understanding. Use of tanning booths increases the risk of skin cancer, so discontinuing use would indicate understanding. Increasing the amount of lean red meat and drinking two glasses of red wine daily are not actions that reduce cancer risk. Increased fiber intake reduces the risk of colon cancer.

The nurse is caring for a client with AIDS who is taking an antiretroviral medication. The client complains of nausea, fever, severe diarrhea, and anorexia. Which of the following medications would the nurse determine to be most effective to relieve the anorexia, as well as to stimulate the client's appetite? Select all that apply. A) Dronabinol (Marinol) B) Zidovudine (Retrovir, AZT) C) Abacavir (Ziagen) D) Ciprofloxacin (Cipro) E) Megestrol (Megace)

Answer: A, E Explanation: A) Megestrol (Megace) and dronabinol (Marinol) are often ordered to increase the client's appetite and promote weight gain. Ciprofloxacin (Cipro) is an anti-infective medication, and zidovudine (Retrovir, AZT) is an antiretroviral agent. Abacavir (Ziagen) is a potent inhibitor of reverse transcriptase.

The nurse is preparing a seminar that discusses the risk and incidence of cancer and culture. What information is considered culturally correct when teaching about the risk of developing cancer? Select all that apply. A) African-American men are more likely to develop prostate cancer than men of other ethnic and racial groups. B) Hispanics have an increased risk of cervical, stomach, and liver cancer. C) The incidence and mortality rate of all type of cancers are lowest in the Caucasian population. D) African-Americans are less likely to develop cancer than any other ethnic or racial group in the United States. E) The Asian/Pacific islander population has the lowest mortality rate of any racial or ethnic group.

Answer: A, E Explanation: A) The incidence and mortality for prostate cancer is highest among African-American men. There is no specific information about the Hispanic population. The incidence and mortality rate for cancer are lower in Native American men and women than in any other ethnic or racial group. African-Americans are more likely to develop cancer than any other ethnic or racial group in the United States. Mortality rates for cancer are the lowest amount the Asian/Pacific Islander population.

The nurse is caring for a client in an allergy clinic. The nurse believes the client is having a reaction to a specific antigen. Which lab test would the nurse assess in order to determine the possibility of a hypersensitivity reaction? A) Indirect Coombs' showing no agglutination B) Patch test with a 1-inch area of erythema C) 2% eosinophils in the WBC count D) Rh antigen with negative results

Answer: B Explanation: A patch test assesses a 1-inch area impregnated with the allergen, which is applied for 48 hours. Absence of a response indicates a negative result. Positive responses are graded from mild (erythema in the exposed area) to severe (papules, vesicles, or ulcerations). Direct Coombs' test detects antibodies in the client's RBC that damage and destroy the cells. This is used following a suspected transfusion reaction to detect antibodies coating the transfused RBCs. This is also part of the crossmatch of a blood type and crossmatch. Indirect Coombs' test detects the presence of circulating antibodies against RBCs. The eosinophil count is 1% to 4%, which is within normal range.

A 1-month-old infant is admitted to the hospital with a temperature of 102°F. Why is this client going to be evaluated for the presence of sepsis? A) Absence of sweat glands B) Immature immune system C) Inadequate red blood cells D) Poor lung elasticity

Answer: B Explanation: A) A child less than 3 months of age with a temperature higher than 100.4°F should be evaluated for sepsis because the child is at increased risk secondary to an immature immune system. The child is not evaluated for sepsis because of the absence of sweat glands, inadequate red blood cells, or poor lung elasticity.

The nurse is caring for a school-aged child who had a bone marrow transplant for the treatment of leukemia several weeks ago. The child requires protective isolation. Which statement by the child's family indicates understanding of this type of isolation? A) "We will encourage oral hygiene twice a day. B) "We will encourage meticulous hand washing among all people in contact with our child." C) "You will have to administer all medications by IM injection." D) "It will be important to restrict all visitors."

Answer: B Explanation: A) A child on protective isolation will be at an increased risk for infection. It will be important to encourage meticulous hand washing among all people who come in contact with the child. Restrict only visitors with colds, flu, or infection. Medications by injection should be avoided. Oral hygiene should be encouraged after every meal.

A nurse working in the Pediatric Intensive Care Unit (PICU) is caring for a child with leukemia. What is the most common type of leukemia in children? A) Chronic lymphocytic leukemia B) Acute lymphocytic (lymphoblastic) leukemia C) Acute myeloid (myeloblastic) leukemia D) Chronic myeloid (myelogenous) leukemia.

Answer: B Explanation: A) Acute lymphoblastic leukemia is the most common type of leukemia in children and the most common cancer affecting children under 5 years of age. The other choices are also types of leukemia, but are incorrect choices.

A child with acute glomerulonephritis experiences blurred vision and headache while in the playroom. Which action should the nurse take at this time? A) Reassure the child and encourage bed rest until the headache improves. B) Obtain the child's blood pressure and notify the physician. C) Check the urine to see if hematuria has increased. D) Obtain serum electrolytes and send a urinalysis to the lab.

Answer: B Explanation: A) Blurred vision and headache may be signs of encephalopathy, a complication of acute glomerulonephritis. The nurse should obtain a blood pressure reading and notify the physician. The physician may decide to order an antihypertensive to bring down the blood pressure. This is a serious complication, and delay in treatment could mean lethargy and seizures. Therefore, the other options (checking urine for hematuria, encouraging bed rest, and obtaining serum electrolytes) do not directly address the potential problem of encephalopathy.

A client prescribed tamoxifen (Nolvadex) for breast cancer treatment asks the nurse how the medication works. What is the best response by the nurse? A) "Tamoxifen works by inhibiting the cellular mitosis of breast cancer." B) "Tamoxifen works by blocking estrogen receptors on breast tissue." C) "Tamoxifen works by binding to the DNA of breast cancer cells." D) "Tamoxifen works by inhibiting the metabolism of breast cancer cells."

Answer: B Explanation: A) Breast cancer is dependent on estrogen for growth. Tamoxifen (Nolvadex) acts by blocking estrogen receptors; the tumor is deprived of estrogen. Tamoxifen does not inhibit the metabolism of breast cancer cells. Tamoxifen does not inhibit the cellular mitosis of breast cancer. Tamoxifen does not bind to the DNA of breast cancer cells.

A client being treated for nephritic syndrome is a vegetarian and has a poor oral intake. Which action should the nurse take to meet this client's nutritional needs? A) Request that the healthcare provider prescribe an appetite stimulant. B) Request a dietician to discuss the client's dietary preferences. C) Encourage the client to eat the food provided on the meal trays. D) Explain that the client will be returning home soon and can resume a preferred diet.

Answer: B Explanation: A) The client is being treated for nephritic syndrome, which could impact the amount of protein permitted to be eaten each day. The client should discuss dietary preferences with a dietician so that the client's nutritional needs will be met. An appetite stimulant is not going to improve the client's appetite if foods are provided that the client does not eat. The nurse should not encourage the client to eat foods that do not support the client's vegetarian diet. The nurse needs to help the client now and not have the client wait to go to home before having foods that the client wants to eat.

A home health nurse has just received the list of clients who need to be seen during the shift. Which client should the nurse plan to see first? A) A client with AIDS who is receiving lamivudine (Epivir) because of a diagnosis of a low CD4 cell count B) A client with Pneumocystis carinii pneumonia (PCP) who called the office this morning to report a new onset of fever, cough, and shortness of breath C) A client with wasting syndrome who has end-stage AIDS who needs modifications and education regarding dietary changes D) A client with a long history of AIDS who is receiving IV antibiotics daily for toxoplasmosis

Answer: B Explanation: A) The home health nurse should see the client with PCP because of the complaint of shortness of breath with the new onset of fever. All of the clients need to be seen by the nurse, but based on the ABCs (airway, breathing, and circulation), the nurse should visit this client first to obtain vital signs and perform a respiratory assessment.

The nurse accompanies the physician into the client's room and listens as the diagnosis of cancer is shared with the client and family. Once the physician leaves the room, the nurse notes that the client and family are teary-eyed regarding the diagnosis. What is the nurse's most appropriate intervention at this time? A) Arrange for the client to complete a medical power of attorney form. B) Provide emotional support in coping with the diagnosis. C) Provide teaching about the treatment options for this form of cancer. D) Help the client and family remain realistic about prognosis.

Answer: B Explanation: A) When a client and family receive a new diagnosis of cancer, it tends to evoke many emotions, including fear, grief, and anger. This is not an opportune time to teach or set goals. The client and family require emotional support at this time, and other actions c

The nurse is caring for a client who is taking an immunosuppressant agent for the treatment of an autoimmune disorder. Which client statement shows that teaching has not been effective? A) "I should drink plenty of water to keep from getting dehydrated." B) "I should drink a lot of grapefruit juice while on these medications." C) "If I experience any joint pain, I should take ibuprofen for the pain as needed every 4 hours." D) "I know to call the physician if I start experiencing a lot of bruising."

Answer: B Explanation: Immunosuppressive agents inhibit T cell development and activation. They are given concurrently with glucocorticoids and in combination with other immunosuppressants, and inhibit immune system activity and organ rejection. Nursing responsibilities include monitoring BUN and creatinine for evidence of nephrotoxicity. The client should avoid grapefruit juice, which can raise cyclosporine levels by 50% to 200% and increase the risk of toxicity. Fluids should be increased to maintain good hydration and urinary output. Ibuprofen is acceptable for immunosuppressive medications, but should not be taken with cytotoxic agents.

The nurse is teaching a group of young parents at the local elementary school health fair about immunity and the importance of vaccination. The nurse is giving the group an example of how active immunity is acquired. Which scenario would provide a client with active immunity? A) Receiving a rabies shot after being bitten by a rabid dog B) Having measles C) Receiving an injection of gamma globulin D) Becoming ill with tetanus and receiving tetanus toxoid

Answer: B Explanation: When the client has the disease, the body stimulates the process of acquired active immunity. Receiving injections for rabies, tetanus, and gamma globulin are examples of artificially acquired passive immunity.

A pediatric nurse is caring for a child with acute lymphoblastic leukemia (ALL). When providing education to the child's parents regarding this disease, the nurse should include: Select all that apply. A) ALL is characterized by abnormal proliferation of all bone marrow elements. B) This form of leukemia is the most common type among children and adolescents. C) Most cases of ALL result from the malignant transformation of B cells. D) This form of leukemia is very rarely seen in children. E) The onset of ALL is usually gradual.

Answer: B, C Explanation: A) Acute lymphoblastic leukemia (ALL) is the most common type of leukemia among children and adolescents. Most cases of ALL result from the malignant transformation of B cells. The onset of ALL is usually acute and rapid. Chronic myeloid leukemia (CML) is characterized by abnormal proliferation of all bone marrow elements.

During a treatment meeting on an oncology unit, the nurse learns that a client is scheduled for chemotherapy before and after surgery. What are the purposes for this client to receive chemotherapy at these specific times? Select all that apply. A) Eradicate all cancer cells. B) Shrink the tumor. C) Kill remaining cancer cells. D) Allow the immune system to kill cancer cells. E) Improve wound healing.

Answer: B, C Explanation: A) It is impossible to eradicate all cancer cells with chemotherapy. Chemotherapy before surgery is used to shrink the tumor. Chemotherapy is used after surgery to kill remaining cancer cells. The use of chemotherapy before and after surgery will not allow the immune system to kill the cancer cells. Chemotherapy is not used to improve wound healing.

The charge nurse for a medical-surgical nursing unit has been notified that a client with tuberculosis is being transported to the unit. Which are the most appropriate actions for infection prevention in this circumstance? Select all that apply. A) Stock the client's supply cart at the beginning of each shift. B) Wear a mask and gown when caring for the client. C) Have the client wear a mask when coming from admissions. D) Perform hand hygiene only after leaving the room.

Answer: B, C Explanation: A) Masks and gowns should be worn when caring for clients who do not reliably cover their mouths when coughing. When a client has an airborne disease and must go elsewhere in the hospital, the client must wear a mask. Supplies to prevent transmission of disease should be stocked at the end of the shift so that adequate supplies will be available for the next healthcare provider. Hand hygiene should be performed before and after client care.

The nurse is caring for a client who is hospitalized on a medical unit for a systemic infection. The client asks the nurse what defenses the body has against infection. The nurse responds that which physiological barrier helps defend the body against microorganisms? Select all that apply. A) Moisturizing the skin B) Adequate urinary output C) Intact skin D) Occasional smoking E) A surgical incision

Answer: B, C Explanation: Voiding a sufficient quantity of urine is a form of barrier protection that helps the body to defend itself against microorganisms. The act of voiding flushes those organisms that might try to enter the body through the urinary meatus. Intact skin is also a physiological barrier that helps defend the body against microorganisms. Occasional smoking does not defend the body from microorganisms; it destroys the cilia in the nose that helps to filter organisms. Moisturizing the skin and a surgical incision can both allow microorganisms to enter the body.

While completing a physical examination, the nurse suspects a client has breast cancer. What did the nurse assess in this client? Select all that apply. A) Rash along the inside of the right arm B) Skin retraction near the left nipple C) Palpable lump in the right axillae D) Flaking skin over the right nipple E) Pain when extending the left arm

Answer: B, C, D Explanation: A) Manifestations of breast cancer include skin retraction in an area of the breast, unusual lump in the underarm region, and flaking skin near the nipple. A rash on the arm and arm pain upon extension are not manifestations of breast cancer.

A 32-year-old client recently diagnosed with multiple sclerosis is a full-time aerobics exercise instructor at a local fitness center. Which statements contain the correct information to give the client when answering her specific questions about lifestyle? Select all that apply. A) "Hyperbaric oxygen treatment is recommended prior to vigorous physical exercise." B) "You will tolerate exercise better in an air-conditioned room." C) "Acupuncture may benefit some of your symptoms." D) "Drinking cold water is recommended during exercise." E) "You will be able to maintain your exercise teaching schedule."

Answer: B, C, D Explanation: A) Symptoms of MS are exacerbated by increased body temperature. Exercising in a cold room and drinking cold beverages keep body temperature down. Acupuncture has low risk and may be beneficial for some symptoms. Hyperbaric oxygen therapy carries more risk than benefit. It is unlikely that a newly diagnosed client with MS will be able to tolerate a full-time exercise instructor role.

The nurse is reviewing the laboratory values of a client who has been newly diagnosed with AIDS. Which laboratory values would the nurse report to the physician? Select all that apply. A) CD4 cell count 1,100/mm3 B) T4 cell count 150 C) CD4 lymphocytes 12% D) Viral load 11,500 copies/mL E) WBC 6,500

Answer: B, C, D Explanation: A) The risk of opportunistic infection is the most common manifestation of AIDS. The risk of opportunistic infection is predictable by the T4 and CD4 cell count. The normal CD4 cell count is greater than 1,000/mm3. All of the labs are abnormal except for the CD4 cell count and the WBC, which was within normal range (4,500-10,000).

The nurse is caring for a thin, older client who was diagnosed with cancer and is receiving aggressive chemotherapy. The client is experiencing severe side effects from the therapy and has lost 10 pounds in the past week. What should the nurse teach the client to do? Select all that apply. A) Purchase fast foods and prepared foods. B) Eat cold foods rather than hot foods, because they are better tolerated. C) Keep a food diary and record intake. D) Eat large frequent meals high in calories. E) Drink liquid supplements to increase intake of nutrients.

Answer: B, C, E Explanation: A) Nutrition is an essential part of caring for all client with cancer but takes on even greater importance in the frail elderly, who may already have nutritional challenges such as poor dentition, inefficient absorption of nutrients, and mediation side effects. The goal of nutritional teaching is to help the client increase caloric and nutrient intake through the use of liquid supplements, small frequent meals, and a food diary that will help the nurse evaluate strengths and weaknesses of the current plan. The client receiving chemotherapy may tolerate cold foods better than hot foods. Fast foods and prepared foods tend to be high in fat and sodium and are not the best choice because they do not contain adequate healthy nutrients. Instead, involving the family in preparing meals or in enrolling in Meals on Wheels may be better options for easy ways of obtaining meals.

A 51-year-old client reports to the nurse an inability to tolerate usual exercise and the feeling of fatigue. The client states that these symptoms have been gradual over time. Which physical assessment findings, along with the client's verbal complaints, would indicate chronic lymphocytic leukemia (CML)? Select all that apply. A) Joint pain B) Pallor C) Splenomegaly D) Abnormal bleeding E) Edema

Answer: B, C, E Explanation: A) The symptoms for CML are insidious and occur over time, affecting older adults. The client may exhibit splenomegaly, pallor, edema, and lymphadenopathy. Bone and joint pain with abnormal bleeding are characteristics of AML, which also occurs in older clients.

A client is admitted with airway edema, bronchoconstriction, and increased mucus production after being exposed to an allergen. What care will the client need to address this inflammation to the respiratory system? Select all that apply. A) Turn and reposition every 2 hours. B) Monitor oxygen saturation. C) Administer oxygen as prescribed. D) Restrict fluids. E) Monitor lung sounds.

Answer: B, C, E Explanation: A) Turning and repositioning every 2 hours would be appropriate to maintain tissue integrity. Monitoring oxygen saturation, administering oxygen, and monitoring lung sounds would be appropriate care for the client with inflammation to the respiratory system. Restricting fluids could cause respiratory secretions to thicken and hinder the client's ability to maintain a clear airway. Fluids should be encouraged.

The healthcare provider has prescribed a client to have peak and trough blood levels drawn to evaluate the therapeutic effect of an IV antibiotic. When should the nurse schedule the blood samples to be drawn? Select all that apply. A) Prior to the discontinuing the antibiotic B) A few minutes before the next scheduled dose of medication C) During the infusion of the antibiotic D) 30 minutes after the IV administration E) 1-2 hours after the oral administration of the medication

Answer: B, D Explanation: A) Antibiotic peak and trough levels monitor therapeutic blood levels of the prescribed medication. The therapeutic range-that is, the minimum and maximum blood levels at which the drug is effective-is known for a given drug. By measuring blood levels at the predicted peak (1-2 hours after oral administration, 1 hour after intramuscular administration, and 30 minutes after IV administration) and trough, usually a few minutes before the next scheduled dose, it is also possible to determine whether the drug is reaching a toxic or harmful level during therapy, increasing the likelihood of adverse effects.

The nurse is planning care for a pediatric client with HIV. The nurse selects Risk for Infection as a priority nursing diagnosis for this client. Which interventions are appropriate for a child with this diagnosis? Select all that apply. A) Give frequent baths to the family dog. B) Teach proper food-handling techniques to the family. C) Provide ample fresh fruits and vegetables to bolster the immune system. D) Assess the health status of all visitors. E) Teach hand-washing techniques to the family.

Answer: B, D, E Explanation: A) A client with HIV is at risk for a myriad of bacterial, viral, fungal, and opportunistic infections because of the effect of the virus on the immune system. The nurse teaches the family to keep those who have symptoms of illness away from the child and also instructs them in proper hand-washing technique and proper food handling to prevent infection. If there are pets, they should be kept outside, as they are a source of infection to the child. Fresh fruits and vegetables are not recommended for a client with a depressed immune system.

A nurse is caring for a client with leukocytosis. Which action by the nurse is most appropriate when caring for this client? A) Instruct the client on the use of an electric razor and soft toothbrush. B) Assess for bleeding and bruising. C) Assess for source of infection. D) Place the patient in reverse isolation precautions.

Answer: C Explanation: A client with leukocytosis has a white blood cell (WBC) count that is elevated above normal (>10,000 mm3). In the presence of an attack such as an infection, additional WBCs are released from the bone marrow, and as WBCs move out of the bone marrow into the blood, the bone marrow increases its production of additional leukocytes, leading to leukocytosis. Instructing the client on the use of an electric razor and soft toothbrush and assessing for bleeding and bruising would be appropriate actions for a client with decreased platelet levels, or thrombocytopenia. Placing the patient in reverse isolation precautions would be appropriate for the client with neutropenia, a decrease in the number of neutrophils.

An elderly client with sepsis has been admitted to the nursing unit. The nurse is planning care and determines that one goal for this client is to maintain normal mental status. Which outcome evaluation implies that the goal has been met? A) The client is agitated. B) The client has a Glasgow coma score of 4. C) The client responds to questions appropriately. D) The client's pupils are fixed and dilated.

Answer: C Explanation: A) A client who responds to questions appropriately is alert and oriented. Restlessness or agitation in a client with sepsis can be a sign that the client's condition is deteriorating. The client with a Glasgow coma score of 4 is comatose. When pupils are fixed and do not respond to light, death is imminent.

A child from a culture other than the nurse's has recently been diagnosed with leukemia. The client's sibling is 6 years old and expressing feelings of anger and guilt. This reaction by the sibling is very upsetting to the parents. How should the nurse explain the sibling's behavior? A) "This behavior is abnormal. I will have the physician refer you to a psychologist." B) "This behavior is just the sibling's way to get attention." C) "This is a normal response. Your other child is also affected by the diagnosis and anger and guilt are expected feelings for a 6-year-old." D) "Your other child should not be so upset. The cancer is easily treated."

Answer: C Explanation: A) A diagnosis of cancer affects the whole family, and initial feelings experienced by the sibling may be anger and guilt. Seldom will the sibling be unaffected; however, the response is not abnormal. Although the sibling may want attention, this is not the best response by the nurse.

The nurse is caring for an African-American client with nephritis. For which long-term complication should the nurse include interventions to prevent when planning this client's care? A) Congestive heart failure B) Diabetes mellitus C) End-stage renal disease D) Hypertension

Answer: C Explanation: A) African-Americans are more likely than Caucasians to progress to end-stage renal disease after nephritis. The nurse should plan interventions to prevent the development of this long-term complication. African-Americans are not more likely to develop congestive heart failure, diabetes mellitus, or hypertension after nephritis.

A nurse is developing a plan of care for a client who was recently diagnosed with HIV. The client admits to being sexually active and states that he will remain sexually active. Which would be a priority nursing diagnosis for this client based on this information? A) Risk for Infection related to immunodeficiency B) Death Anxiety C) Deficient Knowledge related to preventing transmission of HIV D) Social Isolation related to fear of AIDS

Answer: C Explanation: A) All options are potential nursing diagnoses for this client. Deficient knowledge related to preventing transmission of HIV would be the priority diagnosis for this client due to his statement of wanting to remain sexually active. The client will need to be educated on safer sex practices to decrease the risk of transmission to potential sexual partners.

The nurse is caring for a client from India who has extensive deep tissue damage. The nurse notes that the client is also vegan. Which dietary information should the nurse teach this client to enhance the healing process? A) "A low-fat, high-carbohydrate, low-protein diet is best for healing." B) "A high-fat, low-carbohydrate diet is best for healing." C) "A high-carbohydrate, high-protein diet is best for healing." D) "A diet high in protein and vitamin D is best for healing."

Answer: C Explanation: A) Carbohydrates are important to meet the energy demands of healing, and protein is needed is needed for cell growth. The vegan client needs to be taught to eat proteins that provide the essential amino acids that can be lacking in a vegan diet. Fats are needed in moderation for the development of cell membranes. Vitamins necessary to promote healing are C, K, A, and the B-complex vitamins.

The nurse has identified the diagnosis Excess Fluid Volume as appropriate for a client with acute glomerulonephritis. What should the nurse assess to learn the most accurate indication of this client's fluid balance? A) Vital signs B) Intake and output records C) Daily weight D) Serum sodium levels

Answer: C Explanation: A) Daily weight provides the most accurate indication of fluid balance because of the pathophysiology of acute glomerulonephritis. Albumin is lost, causing decreased osmotic pressure and fluid shifting into the interstitial spaces. Accurate weights will provide data to indicate that treatments are effective in pulling fluids from interstitial spaces into the vascular system and then out via kidneys. Intake and output records, serum sodium levels, and vital signs will all provide data indicating fluid balance; however, they are not as accurate as daily weight for determining fluid balance.

A client is experiencing weight gain and foamy dark urine 4 weeks after being treated with antibiotics for a sore throat. About which client statement made during the health history should the nurse provide teaching? A) "I have been trying to get plenty of rest since I have been sick." B) "I have changed to a more nutritious diet." C) "I felt better after 1 week of the antibiotics, so I stopped taking them." D) "I have gained weight in the last 2 weeks."

Answer: C Explanation: A) The client probably had strep throat and did not take the full course of antibiotics, which accounts for the current symptoms that indicate glomerulonephritis. The nurse teaches this client about the importance of taking medications as prescribed to prevent further complications of a disease. Resting when ill is appropriate, as is changing to a more nutritious diet. Gaining weight would support the nurse's suspicion that the client has glomerulonephritis.

The nurse is presenting a talk for the monthly Nursing Case Study education group at her facility. Which client would be a good choice for a case study on multiple sclerosis (MS)? A) Brazilian with chronic parasitic infestation B) Italian with colonized methicillin resistant staphylococcus aureus (MRSA) C) Northern Canadian who has smoked for 25 years D) African-American man in his 20s with a vitamin D deficiency

Answer: C Explanation: A) The client with the greatest risk lives the farthest from the equator and smokes. Smokers are at increased risk of MS. Brazilians and Italians live close to the equator, which lowers the risk of MS. Chronic parasitic infestation lowers the immune response, which lowers the risk of MS. African-Americans and men are at lower risk of developing MS. It is theorized that vitamin D deficiency may increase risk of MS because it is seen less in locales near the equator.

The nurse is assisting the physician with a bone marrow aspiration and biopsy on a client who has leukemia. The client also has thrombocytopenia. When the physician has completed this test, which intervention is a priority for the nurse? A) Dispose of the equipment used, and clean the area properly. B) Label and refrigerate the specimen obtained by the physician. C) Hold pressure on the wound for approximately 5 minutes. D) Make certain the client understands the purpose of the test.

Answer: C Explanation: A) The most important task for the nurse is to prevent bleeding after the biopsy. Holding pressure on the wound for 5 minutes is effective. Dealing with the specimen is accomplished by a third party or after the nurse stabilizes the client. An explanation of the test is performed before the procedure is begun. Cleaning the area is completed after the client is stable and the specimen is sent to the laboratory.

A client being discharged after treatment for nephritis is concerned about her stamina to care for her children once she returns home. Which statement made by the nurse would be most appropriate to address the client's concern? A) "Tell your spouse he has to help you." B) "You will be able to keep up with your family's needs once you return home." C) It sounds like you need some help, so I'll contact Social Services for support." D) "Maybe your children should go and stay with a relative or neighbor for a few weeks."

Answer: C Explanation: A) The nurse should not assume that the client has a spouse. The client will be easily fatigued and will need assistance after treatment for nephritis. The nurse should contact Social Services to obtain support for the client. The client might not have family nearby or neighbors who are available to care for her children for a few weeks.

A client recovering from a hysterectomy does not want to take the prescribed estrogen replacement therapy because of the fear of developing breast cancer. What should the nurse respond to this client? A) "The risk of breast cancer is slightly increased for women who opt to take estrogen replacement therapy." B) "Perhaps you should consider an estrogen-progestin combination therapy." C) "The risk of breast cancer is not increased for women who have had a hysterectomy and take estrogen replacement medications." D) "Taking estrogen replacement is required after a hysterectomy."

Answer: C Explanation: A) The risk for the development of breast cancer is not greater for women who take estrogen replacement therapy after undergoing a hysterectomy. Progestin therapies are not used for women who are in surgical menopause. Further, it is inappropriate for the nurse to make suggestions of a prescriptive nature, as it violates the scope of practice. While it is not mandatory for the client to take estrogen replacement therapy after surgery, the nurse should clarify and correct misconceptions of the client. Estrogen replacement therapy is not associated with breast cancer for women who have undergone a hysterectomy. Taking estrogen after a hysterectomy is optional, not required.

A client being treated with chemotherapy for cancer complains of fatigue, pallor, progressive weakness, exertional dyspnea, headache, and tachycardia. Which diagnosis should the nurse use as the priority when planning this client's care? A) Powerlessness B) Imbalanced Nutrition, Less than Body Requirements C) Activity Intolerance D) Ineffective Coping

Answer: C Explanation: A) The symptoms (fatigue, pallor, progressive weakness, exertional dyspnea, headache, and tachycardia) are caused by aplastic anemia from bone marrow suppression, which is a side effect of the chemotherapy drugs. Decreased red blood cells cause less oxygen to be delivered to body tissues, resulting in tissue hypoxia. Tachycardia is a compensation mechanism to speed up the delivery of oxygen that is available in the fewer number of cells that are present. Tissue hypoxia will result in muscle fatigue, and the symptoms that are related to aplastic anemia will decrease endurance and ability to perform activities. Thus, this NANDA diagnosis should be the first priority. Nutrition or iron deficiency is not the cause of the symptoms, which are related to tissue hypoxia. Powerlessness is the lack of control over current situations, but this is not the client's current problem. Her needs/symptoms are physical, and according to Maslow's theory must be met prior to emotional needs. Although the client might be having coping issues, the physical symptoms are her greatest complaints; therefore, coping is not the top priority in planning her care. Again, physiological needs must be met prior to self-actualization needs.

A client diagnosed with multiple sclerosis has an acute onset of visual changes, fatigue, and leg weakness. The client says that the last time this happened, recovery occurred in a few weeks. Which classification of multiple sclerosis is the client experiencing? A) Progressive-relapsing B) Secondary-progressive C) Relapsing-remitting D) Primary-progressive

Answer: C Explanation: A) There are four classifications of multiple sclerosis. The client has an exacerbation of symptoms and has a history of full recovery. This is classified as relapsing-remitting and is the most common type. Primary-progressive is a steady worsening of the disease with occasional minor recovery. Secondary-progressive begins as relapsing-remitting but the disease becomes worse between exacerbations. Progressive-relapsing is rare, with the disease progressing from the onset with periods of exacerbation.

A 6-year-old male child is being admitted with newly diagnosed acute lymphocytic leukemia. The multidisciplinary team is meeting to plan care for this child and family. Which statement by the parents should receive priority in the nursing planning process? A) "His brother is upset about the amount of time we are away from home." B) "Can we plan a trip out of town sometime this summer?" C) "We are afraid that he will dislodge his central line at school." D) "How do we get our parking validated?"

Answer: C Explanation: A) This is an imminent, potentially life-threatening concern. Financial worries, although a significant concern, would not take precedence over a potentially life-threatening concern. Questions about travel and other family matters should be addressed, but they are not acute issues. The impact of the illness on the client's brother is a realistic concern, but not acute or life-threatening.

The nurse is instructing a group of women between the ages of 40 and 50 about early detection of breast cancer. What should the nurse include in this teaching? A) Perform monthly breast self-exams. B) See a healthcare provider if there is a strong family history of breast cancer. C) Have a yearly mammogram. D) Have a clinical breast exam performed by a healthcare provider every 5 years.

Answer: C Explanation: A) Yearly mammography for all women over the age of 40 is encouraged, as it decreases the mortality from breast cancer. Breast self-exam is no longer recommended for all women. The American Cancer Society recommends that young women who choose to do breast self-exams have their technique validated by a healthcare practitioner at a yearly physical exam. The earlier a lump is discovered, the greater the effectiveness of treatment. Discussing a family history of breast cancer would be part of the annual breast exam performed by a healthcare provider. It is inappropriate for women in this age group to have a clinical breast exam every 5 years.

A preschool-age child is being seen in a pediatric oncology clinic. The nurse assigned to care for the client anticipates a diagnosis of cancer. Which reaction is considered common for the preschool-age child to experience with illnesses and hospitalizations? Select all that apply. A) Unawareness of the illness and its severity B) Understanding of what cancer is and how it is treated C) Thoughts that they caused their illness and are being punished D) Confusion as to why a parent is unable to make the illness go away E) Acceptance, especially if able to discuss the disease with children their own age

Answer: C, D Explanation: A) Preschool-age children are egocentric and have magical thinking, and thus might believe they caused their own illness. This age group may also be confused as to why their parents cannot make the illness go away. Immediate acceptance will not occur with children of any age. Adolescents find contact with others who have gone through their experience helpful. School-age children can understand a diagnosis of cancer. Infants and toddlers are unaware of the severity of the disease.

A nurse is educating a group of pregnant clients regarding infant immunity. Which statements will the nurse include in the teaching? Select all that apply. A) IgM is the only immunoglobulin that crosses the placental barrier. B) IgA and IgE are present at birth. C) In the infant, maternal IgG disappears by 6-8 months. D) Infants and children have differing amounts of some immunoglobulins. E) A newborn's levels of IgG differ widely from those of the mother.

Answer: C, D Explanation: In regard to newborn and infant immunity, IgG is the only immunoglobulin that crosses the placental barrier. Because of this, a newborn's level of IgG is similar to that of the mother. The maternal IgG disappears by the time the infant is 6-8 months old. IgA and IgE are not present at birth. Infants and children have differing amounts of some immunoglobulins.

The nurse in a rheumatology clinic is managing care for clients who receive nonsteroidal anti-inflammatory drugs (NSAIDs) for the treatment of their disease process. What are the primary laboratory tests the nurse will assess prior to initiation of therapy? Select all that apply. A) Serum amylase B) Electrolytes C) Creatine clearance D) Complete blood count (CBC) E) Liver function tests

Answer: C, D, E Explanation: A) It is important to assess the client's creatine clearance to determine kidney function prior to initiation of nonsteroidal anti-inflammatory drug (NSAID) therapy. It is also important to assess the client's liver function tests and complete blood count (CBC) prior to initiation of NSAID therapy. There is no need to assess the client's electrolytes or serum amylase; these are not factors that are affected by NSAIDs.

The nurse is caring for a client who has experienced a sports-related injury to his knee. During the morning assessment, what signs of inflammation will the nurse most likely assess? Select all that apply. A) Pitting edema B) Pallor C) Swelling D) Warmth E) Pain

Answer: C, D, E Explanation: A) Swelling, warmth, and pain are signs of inflammation. Pallor is not a sign of inflammation; redness is. Pitting edema is not a sign of inflammation.

A nurse is caring for a client with tuberculosis who is taking Rifampin for treatment of the disease. Of which nursing considerations should the nurse be aware regarding this medication? Select all that apply. A) Administer with meals to reduce gastrointestinal side effects. B) Record a baseline visual examination before therapy. C) Administer on an empty stomach. D) Administer by deep intramuscular injection into a large muscle mass. E) Monitor CBC, liver function studies, and renal function studies for evidence of toxicity.

Answer: C, E Explanation: A) Rifampin is an oral antituberculosis medication that should be administered on an empty stomach. The nurse should monitor the CBC, liver function studies, and renal function studies. A baseline visual examination before therapy is necessary with ethambutol, another antituberculosis medication.

The nurse identifies the diagnosis of Ineffective Peripheral Tissue Perfusion as being appropriate for a client with septicemia. Which intervention will address this client's health problem? A) Monitor heart rate every hour. B) Assess temperature every 4 hours. C) Monitor pupil reactions every 8 hours. D) Monitor for cyanosis.

Answer: D Explanation: A) Assessing temperature and monitoring heart rate and pupil reaction are important when assessing a client with septicemia, but a change in skin color will alert the nurse immediately of decreased tissue perfusion.

The nurse is caring for a client who has just been diagnosed with chronic myeloid leukemia (CML). The client and the nurse are discussing the anticipatory grieving process. Which action by the nurse would be inappropriate at this time? A) Make referrals for support or bereavement groups. B) Identify family stress management strategies. C) Encourage the client to see an attorney now to get affairs "in order" before it is too late. D) Encourage the client to share feelings and discuss grieving.

Answer: D Explanation: A) Encouraging the client to get affairs "in order" now to avoid waiting until it is too late is not appropriate at this time: Although this topic is helpful to prepare for the actual death, this is not the time because this removes all hope. Establishing open communication and sharing of feelings to discuss grieving is appropriate at this time: The nurse should establish a rapport and use therapeutic communication to allow the client to express feelings and emotions about the new diagnosis of CML. Making referrals for support or bereavement groups is appropriate at this time: Offering information and resources about agencies that deal with grieving is an option to show the client that agencies can assist when the need is felt or when the client is ready to use them. In addition, this helps the client understand that anticipatory grieving is a normal process that occurs. Identifying family stress management strategies is appropriate at this time: Exploring possible stressors and strategies associated with the disease progression will give the client a realistic approach to understanding the disease process and its consequences. This also helps the client begin to share with the family to build a foundation for mutual understanding and trust.

An occupational health nurse is screening a new employee in a long-term care facility for tuberculosis. The employee questions why purified protein derivative (PPD) testing is done twice. Which is the most appropriate response by the nurse? A) "Different medication is used in the second PPD." B) "The treatment for TB is 6 months of medication, and we want to make sure the first results of the first PPD were accurate." C) "The first PPD was not interpreted in the correct time frame of 48-72 hours." D) "There is an increased risk for a false-negative response for people who work in long-term care facilities. The two-step is recommended to accurately screen for TB."

Answer: D Explanation: A) PPD testing is done in a two-step process for people who work in long-term care facilities because of the risk of false-negative responses. Treatment for TB for 6 months is not a reason to complete the PPD twice. PPD testing is not done twice because different medication is used. Evaluating the test at the wrong interval is not the reason that the PPD is done twice for long-term care facility employees.

An elderly client who resides in a long-term care facility is admitted to the hospital with sepsis. The family is tearful and does not understand how their family member got so sick from a bed sore. What should the nurse teach the family to help in the care of the client? A) Alert the staff when the IV runs dry. B) Help the nurse with dressing changes. C) Assist the client to the bathroom so there is not a fall. D) Assist the client with meals to obtain optimal nourishment.

Answer: D Explanation: A) Poor nutritional status and a slow-functioning immune system contribute to the risk for sepsis in the older client. The family can help by assisting and encouraging the client to take in the proper nourishment needed. The family is not responsible for IV or dressing changes. Assisting the client to the bathroom should be done by the staff who are responsible for client safety.

The nurse is caring for a client who is receiving multiple drugs for treatment of tuberculosis. The nurse teaches the client the rationale for the multiple-drug treatment and evaluates learning as effective when the client makes which statement? A) "Multiple drugs are necessary to develop immunity to tuberculosis." B) "Multiple drugs are necessary because I became infected from an immigrant." C) "Multiple drugs will be required as long as I am contagious." D) "Multiple drugs are necessary because of the risk of resistance."

Answer: D Explanation: A) Tuberculosis bacilli are likely to develop resistance to one drug, so multiple drugs must be used. Treatment must be continued long after the client is no longer contagious. There is no indication that the client contracted TB from an immigrant. Multiple drugs are used for all cases of TB.

A new graduate nurse is performing an admission assessment on a client with symptoms that indicate the client may have HIV. Which question does the nurse ask to identify a major risk factor for contracting HIV? A) "Has your partner been experiencing these symptoms?" B) "Do you always practice safe sex?" C) "Have you had any fever, diarrhea, or chills over the last 48 hours?" D) "Have you ever experimented with intravenous drugs?"

Answer: D Explanation: A) Use of recreational drugs, such as intravenous drugs, is a risk factor for contracting HIV. Asking about safe sexual practices is important, but intravenous drug use would put the client at greater risk for contracting HIV. The nurse cannot ask about the partner without the partner's consent; however, if the partner were present and positive, it would be a risk factor. Recent symptoms are not a risk factor.

The nurse is caring for a client being seen at an urgent care clinic because of an infected arm. The client tells the nurse he was bitten by a raccoon on a recent camping trip. The nurse expects treatment for this client to include which of the following? A) An injection of immunoglobulin B) A tetanus toxoid injection C) Mother's breast milk with antibodies in it D) An immunization for rabies

Answer: D Explanation: Receiving an immunization for rabies is an example of artificially acquired passive immunity. Receiving tetanus and immunoglobulin are also examples but would not be used in the case of an animal bite. Mother's breast milk is another example of passive immunity, but would not be used in the case of an animal bite.

The nurse is caring for a client with severe inflammation. Which assessment findings would indicate a systemic reaction to inflammation? Select all that apply. A) Erythema B) Edema C) Pain D) Tachypnea E) Tachycardia

Answer: D, E Explanation: A) If the nurse observes a systemic reaction, the client will exhibit manifestations including temperature, increased pulse, tachypnea, and leukocytosis. Erythema, warmth, pain, edema, and functional impairment indicate a local reaction.


Related study sets

Microbiology Chapter 9: Exchange of DNA between Bacteria

View Set

Which of the following is not a macromolecule?

View Set

NTE CHAPTER 17: MONOMER LIQUID & POLYMER POWDER NAIL ENHANCEMENTS

View Set

Principles of Marketing Midterm Study Guide

View Set

Reproductive Physiology 3, Ovary and Follicular Development

View Set

Chapter 3- The Aquatic Environment

View Set